Question about vector fields, div, curl grad

Click For Summary
The discussion centers on proving two key vector calculus identities: if the divergence of a vector field X is zero, then X can be expressed as the curl of another vector field Y; and if the curl of X is zero, then X can be expressed as the gradient of some scalar field Y. Participants suggest using the divergence theorem and Stokes' theorem to establish these proofs, with one noting that the second proof is simpler. The conversation references Helmholtz's theorem, which relates to these identities in the context of electromagnetism. Overall, the thread seeks guidance on finding formal proofs for these mathematical statements.
bobkolker
Messages
14
Reaction score
3

Homework Statement



I need a pointer to a proof of the following items:
if div X =0 then X = curl Y for some field Y.
if curl X = 0 then X = grad Y for some field Y.

Can anyone provide a pointer to a proof?

Thanks.

Bob Kolker

Homework Equations

The Attempt at a Solution

 
Physics news on Phys.org
Here are a couple of hints

First one: think about what X would look like on the boundary of a sphere
Second one: think about what X would look like on the interior of a closed plane curve (a loop that lies in a single plane)
 
I have no doubt the equations are true. I am looking for a reference to a proof. Can you help me out? :Thanks.
 
Unfortunately I don't have a good reference for you, but I remember these two proofs. They both follow a similar procedure using the divergence theorem and Stokes theorem around arbitrary orientable closed surfaces and closed plane curves respectively. Start with the second one, it's a little simpler.
 
I think this is called Helmholtz's theorem in E&M (Electricity and Magnetism). The div(curl A)=0 in all cases and also curl (grad V)=0 in all cases, but the converse that there exists a field, etc. is Helmholtz's theorem.
 
Last edited:
Question: A clock's minute hand has length 4 and its hour hand has length 3. What is the distance between the tips at the moment when it is increasing most rapidly?(Putnam Exam Question) Answer: Making assumption that both the hands moves at constant angular velocities, the answer is ## \sqrt{7} .## But don't you think this assumption is somewhat doubtful and wrong?

Similar threads

  • · Replies 6 ·
Replies
6
Views
2K
Replies
3
Views
3K
  • · Replies 11 ·
Replies
11
Views
4K
  • · Replies 8 ·
Replies
8
Views
2K
  • · Replies 1 ·
Replies
1
Views
2K
Replies
4
Views
2K
Replies
4
Views
2K
  • · Replies 2 ·
Replies
2
Views
2K
  • · Replies 4 ·
Replies
4
Views
2K
  • · Replies 2 ·
Replies
2
Views
3K